LSAT and Law School Admissions Forum

Get expert LSAT preparation and law school admissions advice from PowerScore Test Preparation.

User avatar
 Dave Killoran
PowerScore Staff
  • PowerScore Staff
  • Posts: 5852
  • Joined: Mar 25, 2011
|
#85368
Complete Question Explanation
(The complete setup for this game can be found here: lsat/viewtopic.php?f=315&t=9170)

The correct answer choice is (B)

The conditions imposed by the question stem produce the following sequence:

G1-Q6-d1.png

Because there are three investors ahead of F, and therefore the earliest day F could view the site is day 4, from the first rule we can infer that F must view the site on day 5.

Because H, G, and L must all view the site before F, an inference can made that either H, G, or L must view the site on day 3. If H, G, and L do not view the site on day 3, then H and G view the site on day 1 and 2, respectively, and the only remaining day for L to view the site is day 4, a violation of the second rule. Among the answer choices, only answer choice (B) lists one of the three investors H, G, or L, and therefore answer choice (B) is correct.
You do not have the required permissions to view the files attached to this post.

Get the most out of your LSAT Prep Plus subscription.

Analyze and track your performance with our Testing and Analytics Package.